Vous êtes sur la page 1sur 71

STEP Mark Scheme 2015

Mathematics
STEP 9465/9470/9475
October 2015

This mark scheme is published as an aid to teachers and students, to indicate the
requirements of the examination. It shows the basis on which marks were awarded
by the Examiners and shows the main valid approaches to each question. It is
recognised that there may be other approaches and if a different approach was
taken in the exam these were marked accordingly after discussion by the marking
team. These adaptations are not recorded here.
All Examiners are instructed that alternative correct answers and unexpected
approaches in candidates scripts must be given marks that fairly reflect the relevant
knowledge and skills demonstrated.
Mark schemes should be read in conjunction with the published question papers and
the Report on the Examination.
The Admissions Testing Service will not enter into any discussion or correspondence
in connection with this mark scheme.

UCLES 2015
More information about STEP can be found at:
www.stepmathematics.org.uk

Contents
STEP Mathematics (9465, 9470, 9475)
Mark Scheme
STEP Mathematics I
STEP Mathematics II
STEP Mathematics III

Page
4
18
45

SI-2015/Q1
(i)
y = ex(2x 1)(x 2)
( 12 , 0) & (2, 0)
dy
= ex(2x2 x 3)
dx
= ex(2x 3)(x + 1)

( 32 , e1.5) & (1, 9e 1)

B1
B1

Correct factorisation of quadratic term (or formula, etc.)


Noted or shown on sketch

M1

Derivative attempted and equated to zero for TPs

A1 A1 Noted or shown on sketch


(if y-coords. missing, allow one A1 for 2 correct x-coords.)

G1

Generally correct shape

G1

for (0, 2) noted or shown on sketch

G1

for negative-x-axis asymptote


(penalise curves that clearly turn up away from axis
or that do not actually seem to approach it)

Give M1 for either 0, 1, 2 or 3 solutions OR clear indication they know these arise from where a
horizontal line meets the curve (e.g. by a line on their diagram) implied by any correct answer(s)
Then y = k has

NO solutions for k < e1.5

A1

ONE solution for k = e1.5 and k > 9e 1

A1 A1

TWO solutions for e1.5 < k 0 and k = 9e 1

A1 A1

THREE solutions for 0< k < 9e 1

A1

FT from their y-coords.of the Max. &Min. points.

(ii)

G1

Any curve clearly symmetric in y-axis

G1

Shape correct

G1

A Max. TP at (0, 2) FT

G1

Min. TPs at (

3
2

, e1.5) FT

G1

Zeroes at x =

1
2

, 2 FT

SI-2015/Q2
(i)
M1

(ii)

Use of cos(A B) formula with A = 60o, B = 45o OR A = 45o, B = 30o


or 2 cos2 15o 1 etc.

3 1

A1

Exact trig.values used (visibly) to gain cos 15o =

M1

Similar method OR sin = 1 cos 2 (as 15o is acute, no requirement to justify +vesq.rt.)

A1

sin 15o =

M1
A1
A1

Use of cos(A + B) formula and double-angle formulae OR de MoivresThm. (etc.)


cos3 4cos3 3cos
Justifying/noting that x = cos is thus a root of 4 x 3 3 x cos 3 0

3 1
2 2

2 2

legitimately (Given Answer)

(however legitimately obtained)

M1

For serious attempt to factorise 4 x 3 c 3 3 x c as linear quadratic factors

A1

or via Vietas Theorem (roots/coefficients)


( x c) 4 x 2 cx c 2 3

M1


Solving 4 x 4cx 4c 3 0 FT their quadratic factor
Remaining roots are x = c c 4c 3
2

1
2

M1

Use of s 1 c 2 to simplify sq.rt. term

A1

x=

1
2

cos

3 sin

(iii)

M1

1 3 3
2
y y
0
2
2
2

A1

1
4
2

M1

cos3 =

A1

= 15o
1
y cos ,
2

M1
A1
A1
A1

1
y 3

y
0
2

2
= cos 45o
2
1
2

y = 2 cos 15o =

cos

cos

3 sin ,

1
2

3 sin with their

3 1
2

3 sin 15 o cos15 o =

3 1
2

3 sin 15 cos15 = 2
o

SI-2015/Q3

1
2

(b a)

P
1
2

(b a)

B1
M1
A1

For correct lengths in smaller


By similar s (OR trig.ORcoord.geom.)
PQ 12 (b a)
b(b a)
1
PQ =
b
ba
2 (b a )

M1

so a guard at a corner can see 2(b + PQ)

A1

4b 2
4ba
=
(might be given as all but
or as a
ba
ba
fraction of the perimeter)

Lengths
M1

1
2

A1

1
2

a and 12 (b a) in smaller

By similar s (OR trig.ORcoord.geom.)


1
1
b
a
b(b a )
2
1 2
PQ =
PQ 2 (b a)
2a

M1

so a guard at a midpoint can see b + 2PQ

A1

b2
b( 4a b)
(might be given as all but
or as a
a
a

fraction of the perimeter)

Lengths
P

M1
A1

M1
A1

1
2

a and 12 (b a) in smaller

By similar s (OR trig. OR coord.geom.)


1
a
PQ
ba
1 2
PQ =
b
ba
2 (b a )
so a guard at a midpoint can see 4b 2PQ
2ba
2b(2b 3a )
=
(might be given as all but
or as a
ba
ba
fraction of the perimeter)

B1

Recognition that b = 3a is the case when guard at M / C equally preferable


(P at corner in the two M cases)

4b 2 b 2
b2
3a b

b a a a(b a)
Correct conclusion: Guard stands at C for b< 3a and at M for b> 3a

M1A1 Relevant algebra for comparison of one case


A1

4b 2 2b(2b 3a )
2ba
3a b

ba
ba
(b a )(b a )
Correct conclusion: Guard stands at C for b< 3a and at M for b> 3a

M1A1 Relevant algebra


A1

Overall, I am anticipating that most attempts will do the Corner scenario and one of the Middle scenarios. This
will allow for a maximum of 12 = 5 (for the Corner work) + 4 (for the Middle work) + 3 (for the comparison).
In this circumstance, it wont generally be suitable to give the B1 for the b = 3a observation.

SI-2015/Q4
M1

When P is at (x,

1
4

x 2 ) ... and makes an angle of with the positive x-axis

A1

... the lower end, Q, is at x b cos ,

M1

Also, y 14 x 2

A1

x = 2 tan

dy 1
x = tan
dx 2

i.e. P = 2 tan , tan 2

1
4

x 2 b sin

A1A1 so thatQ = 2 tan b cos , tan b sin


2

obtained legitimately (Given Answer)

2 tan
cos

M1A1

When x = 0, 2 tan b cos b

M1A1

Substg. into y-coordinate yA = tan 2 2 tan

sin
tan 2
cos

M1A1

Eqn. of line AP is y x tan tan 2

M1A1

Area between curve and line is

B1

Correct limits (0, 2tan )

A1A1

1
12

A1A1

2
3

tan 3 2 tan 3 2 tan 3

A1

2
3

tan 3 obtained legitimately (Given Answer)

x 3 12 x 2 tan x tan 2

1
4

x 2 x tan tan 2

dx

(Any 2 correct terms; all 3)


(Any 2 correct terms; all 3 FT)

ALTERNATIVE
O

A
M1 A1

for obtaining the conversion factor bcos = 2 tan or tan2 = 12 b sin

M1 A1

for distances OB = BC (= 12 b cos ) and so PC = OA = tan2

M1 A1

giving OABCPB

A1

Area is

B1
A1 A1

Correct limits (0, 2 tan ) used


Correct integration; correct Given Answer

ALTERNATIVE

1
4

x 2 dx

Translate whole thing up by tan2 and calculate

b cos
1
4
0

x 2 tan 2 dx

SI-2015/Q5
(i)

M1A1

(t 1) x 3
f(x) =

x 1

A1

M1

Differentiating by use of Quotient RuleOR taking logs.anddiffg. implicitly)


d x
for
2 2 x. ln 2 seen at any stage
dx

B1
A1
A1

2x
x

x .2 x . ln 2 2 x
dy
=
dx
x2
1

TP at
, e ln 2
ln 2

(y-coordinate not required)

B1

Jusitfying that the TP is a minimum

G1
G1

Generally correct -shape


Asymptotic to y-axis
and TP in FT correct position

(ii)

M1
A1
B1
M1A1
A1

Let u 2 1 x 2 2 xt
2u du = 2xdt
t: (1, 1) u: ( | 1 + x |, | 1 x | ) Correct limits seen at any stage
1
Full substn. attempt; correct g(x) =
1 . du
x
1
g(x) = 1 x 1 x
n. may be done directly, but be strict on the limits
x
2
x 1
x

org(x) = 2 1 x 1

2
x 1
x
(Must have completely correct three intervals: x< 1, 1 x 1 , x> 1)

M1
A1 A1

Graph split into two or three regions


Reciprocal graphs on LHS & RHS (must be asymptotic to x-axis)

A1

Horizontal line for middle segment

(Allow even if they approach y-axis also)

SI-2015/Q6

Let P, Q, R and S be the midpoints of sides (as shown)


Then
M1A1
p= 12 a 12 b , q = 12 b 12 a ,

r= 12 a 12 b , s = 12 b 12 a

and
M1A1
A1
A1

a a
QR PS 12 b b
PQ SR

1
2

so that PQSR is a //gm.


(opposite sides // and equal)

M1
A1

PQ QR PQ QS
=

1
4

1
2

a a 12 b b

a b a b a b a b

for use of the scalar product


Do not accept a b etc.

M1

Use of perpendicularity of OA, OB and OA, OB


= 14 a b a b

M1
A1

AOB = AOB = 180o ; and cos(180o ) = cos


= 0 since a b ab cos and a b a b cos
and we are given that a = b and a = b
so that PQRS is a rectangle (adjacent sides perpendicular)

A1

( a ) 2 2a a

B1

PQ 2 SR 2 PQ PQ =

B1

QR 2 PS 2 =

M1

Since a = b, a = b and a a aa cos 90 o , b ab bb cos 90 o

A1

it follows that PQRS is a square (adjacent sides equal)

1
4

1
4

(b ) 2 2b b

( a ) 2 2aa cos 90 o

M1A1

Area PQRS =

M1

... which is maximal when cos 90 o 1

A1

i.e. when 90 o

1
4

SI-2015/Q7
M1

f (x) = 6ax 18x2


= 6x(a 3x)
= 0 for x = 0 and

A1A1

f(0) = 0

A1A1

f( 13 a) = 19 a3

(Min. TP)

A1

x = 13 a

(Max. TP)

since f(x) is a negative cubic

(f(0) = 0 and the TPs may be shown on a sketch award the marks here if necessary)

Evaluating at the endpoints


f( 13 ) = 19 (3a + 2); f(1) = 3a 6

M1
A1A1

1
9

M1

(3a + 2) 19 a3 a3 3a 2 0

(a + 1)2(a 2) 0
and since a 0, a 2

M1
A1

1
9

M1

a3 3a 6 a3 27a + 54 0

(a 3)2(a + 6) 0
which holds for all a 0

M1
A1

1
9

M1

(3a + 2) 3a 6 3a + 2 27a 54
8(3a 7) 0
a 73

A1

(which, actually, affects nothing, but working should appear)

Thus
B1B1B1

19 (3a 2) 0 a 2

M(a) = 19 a 3
2a3
3a 6
a3

(Ignore non-unique allocation of endpoints)

(Do not award marks for correct answers unsupported or from incorrect working)

SI-2015/Q8
(i)
M1
M1
A1

S = 1 + 2 + 3 + + (n 2) + (n 1) + n
S = n + (n 1) + (n 2) + + 3 + 2 + 1
2S = n (n + 1)
S = 12 n(n + 1) obtained legitimately (Given Answer)

Method
Adding

(Allow alternatives using induction or the Method of Differences, for instance, but NOT by stating
that it is an AP and just quoting a formula; ditto -number formula)

(ii)
M1A1
E1

(N m)k + mk (k odd)
k
k
k k 1
m N m k m k
= N k mN k 1 m 2 N k 2 ...
1
2
k 1
which is clearly divisible by N (since each term has a factor of N)
(Allow alternatives using induction, for instance)

Let S = 1k + 2k + + nk

an odd no. of terms

M1

= 0k + 1k + 2k + + nk

M1

= (n 0) k 0 k ( n 1) k 1k ... ( 12 n 12 ) k ( 12 n 12 ) k

an even no. of terms

(no need to demonstrate final pairing but must explain fully the pairing up or the single extra nk term)

E1

and, by (ii), each term is divisible by n.

For

S = 1k + 2k + + nk

an even no. of terms

M1

= 0k + 1k + 2k + + nk

M1

= (n 0) k 0 k (n 1) k 1k ... ( 12 n 1) k ( 12 n 1) k 12 n

an odd no. of terms

(no need to demonstrate final pairing but must explain the pairing and note the separate, single term)

E1

and, by (ii), each paired term is divisible by n


and the final single term is divisible by 12 n required result

M1
A1
E1

By the above result for n even, so that (n + 1) is odd


(n + 1) | 1k + 2k + + nk + (n + 1)k
(n + 1) |S + (n + 1)k (n + 1) |S

M1
A1

By the above result for n odd, so that (n + 1) is even


k
k
k
k
1
2 (n + 1) | 1 + 2 + + n + (n + 1)

E1

1
2

(n + 1) | S + (n + 1)k

1
2

(n + 1) |S (as

1
2

(n + 1) is an integer)

E1

Since hcf(n, n + 1) = 1 hcf( 12 n, n + 1) = 1 for n even

E1

and hcf(n,
So it follows that

1
2

1
2

(n + 1)) = 1 for n odd

n(n + 1) | S for all positive integers n

SI/15/Q9
M1

Time taken to land (at the level of the projection) (from y = utsin
is t

A1
M1
A1
M1A1
A1

Bullet fired at time t 0 t


lands at time
6

2u

TL t sin t
g
3

1
dTL
2 u

1
cos t = k cos t
k
g
dt
3

= 0 when k cos t
3

2u 2 sin cos
(from y = utsin
g

Horizontal range is R

A1

RL

M1A1

d 2TL
22 u

sin t 0 maximum distance


2
g
dt
3

M1A1

0t

A1
M1
A1
M1A1

gt2 , y = 0, t 0)

2u sin
(may be implicit)
g

M1A1

M1

1
2

2u 2
k 1 k 2
g

1
2

gt2 with above time)

obtained legitimately (Given Answer)

1
3

in k cos t k
6
2
2
3

dTL
0 throughout the guns firing
dt
and TL is a (strictly) decreasing function.
Then TL max. occurs at t = 0
If k <

1
2

then

i.e.
and RL

2u 1
3 u2 3

g 2 2
2g

SI-2015/Q10
B1

Speed of rain relative to bus is vcos u (or u vcos if negative)

M1A1

When u = 0, A hvcos + avsin (width of bus and time units may be included as factors)

E1

When vcos u > 0, rain hitting top of bus is the same, and rain hits back of bus
as before, but with vcos u instead of vcos
When vcos u < 0, rain hitting top of bus is the same, and rain hits front of bus
as before, but with u vcos instead of vcos
Together, A h |vcos u| + avsin
Fully justified (Given Answer)

E1
A1

M1
A1
M1

1
so we need to minimise
u
av sin h | v cos u |
J

(Ignore additional constant-of-proportionality factors)


u
u

Journey time

For vcos u > 0,


if w vcos , we minimise J

av sin hv cos

h
u
u

E1
E1

and this decreases as u increases


and this is done by choosing u as large as possible; i.e. u = w

M1

For u vcos > 0,

av sin hv cos

h
u
u
and this decreases as u increases if a sin > hcos
so we again choose u as large as possible; i.e. u = w

we minimise J
E1
E1

[Note: minimisation may be justified by calculus in either case or both.]

M1
A1

If a sin < hcos , then J increases with u when u exceeds vcos


so we choose u = vcos in this case

M1A1

M1
A1

If a sin = hcos then J is independent of u, so we may as well take u = w

av sin hv cos

h
u
u
Which always decreases as u increases, so take u = w again
Replacing by 180o gives J

SI-2015/Q11
(i)

B1

O1: F = F1
O2: F = F2

O1
(Both, with reason)

O2

(ii)

B1

Resg. ||plane (for C1):

F1 R W1 sin

B1

Resg.r. plane (for C1):

R1 F W1 cos

B1

Resg.||plane (for C2):

F2 R W2 sin

B1

Resg.r. plane (for C2):

R2 F W2 cos

Max 4 marks to be given for four independent statements (though only 3 are required).
One or other of
Resg.||plane (for system) : F1 F2 W1 W2 sin
Resg.r. plane (for system) : R1 R2 W1 W2 cos
may also appear instead of one or more of the above.
(F1 and F2 may or may not appear in these statements as F, but should do so below)

FR FR
using and

W1
W2

M1A1

Equating for sin :

M1A1

W W2
R
Re-arranging for F in terms of R: F 1
W1 W2

M1

Use of the Friction Law, F R

A1

W1 W2

W1 W2

obtained legitimately (Given Answer)

M1A1

M1A1

(e.g.) tan

W W2

F F 1
W1 W2
W W2

F
=
using R 1
W

W
R1 F
2
1

Substg. for R

2W1

F
W
W

2
= 1
R1 F1

A1
M1A1

FR
R1 F

Substg. for R1 (correct inequality) using Friction Law F1 1 R1 R1

F1

2W1

F
W1 W2

F1
F1

M1

A1

Tidying-up algebra

2W1

F
W1 W2

=
1 1

F
1

tan

2 1W1
obtained legitimately (Given Answer)
1 1 W1 W2

SI-2015/Q12
r

(i)

M1A1

n 1 3
P(exactly r out of n need surgery) =
r 4 4

nr

(A binomial prob. term; correct)

(ii)

M1
B1B1A1

nr

M1

e 8
r!

8n
1 3

n r (n r ) ! 4 4

M1

e 8
r!

8n
3n r

Attempting to deal with the powers of 3 and 4

4n
n r (n r ) !

A1

e 8
=
r!

M1

e 8 2 r
r!

6n r
Splitting off the extra powers of 2 ready to

n r (n r ) !

M1

e 8 2 r
r!

A1

(iii)

nr

e 8 8 n
n!
1 3

Attempt at sum of appropriate product terms

r ! (n r ) ! 4 4
nr n!
Limits
All internal terms correct; allow nCr for the A mark

P(S = r) =

Factoring out these two terms

2 n 3n r
Correctly

n r (n r ) !

m0

e 8 2 r
e6
r!

6m
adjust the lower limit (i.e. using m = n r)
m!

i.e.

e 2 2 r
r!

A1

which is Poisson with mean 2

M1

P(M = 8 |M + T = 12)

Identifying correct conditional probability outcome

A1A1A1

e 2 28 e 2 2 4

8!
4!
=
e 4 412
12 !

One A mark for each correct term (& no extras for 3rd A mark)

A1A1

212 12 !
412 8 ! 4 !

(Give B1 for noting this without the working)

Powers of e cancelled; factorials in correct part of the fraction


(unsimplified is okay at this stage)

A1

495
4096

SI-2015/Q13
Reminder
A : the 1st6 arises on the nth throw
B : at least one 5 arises before the 1st6
C : at least one 4 arises before the 1st6
D : exactly one 5 arises before the 1st6
E : exactly one 4 arises before the 1st6

56 n 1 16

(i)

M1A1

P(A) =

(ii)

M1A1

By symmetry (either a 5 or a 6 arises before the other), P(B) =

(iii)

M1

The first 4s, 5s, 6s can arise in the orders 456, 465, 546, 564, 645, 654

A1

P(BC) =

1
3

1
2

(i.e. by symmetry but with three pairs)

(iv)

M1A1A1

P(D) =

16 16 16 64 16 16 64 2 16 ...
2

1
M1 for infinite series with 1stterm ; A1 for 2nd term ; A1 for 3rd term and following pattern

M1

361 1 2 23 3 23 2 ...

For factorisation and an infinite series

M1

361 1 23 2

Use of the given series result

A1

1
4

(v)

M1

P(DE) = P(D) + P(E) P(DE)

Stated or used

B1

P(E) = P(D) = answer to (iv)

Stated or used anywhere

M1A1A1

P(DE) =

62 16 16 63 62 16 16 63 2 62 16 16 ...
3

M1 for infinite series with 1 term ; A1 for 2nd term ; A1 for 3rd term and following pattern
st

M1

1081 1 3 12 6 12 2 ...

For factorisation and an infinite series

M1

1081 1 12 3

Use of the given series result

A1

P(DE) =

1
2

272

23
54

Question1
(i)

ln 1

For

Therefore

If
0,
Therefore

ln 1
ln 1

Therefore

ln 1

So,

ln 1

So,

ln 1

B1

ln

ln

ln

ln

M1

M1

ln 1

A1

ln 1

Therefore

If
0,
Therefore

So,

Therefore

B1

M1

ln 1

0for0

ln 1
ln 1

1.

0
isnegativefor0

ln 1

0forall
1

ln 1
1

1,

ln 1

B1

1.
1.

B1

ln

A1

ln

2 ln

ln

M1M1
A1

So,
ln 1
As ,ln
Therefore,
1

1
1

ln 2

ln

ln
1

ln

M1A1

1 0

B1

A1

B1

ln

ln

A1

0forallpositive .

For0

0
ispositiveforallpositive .

ln

0for

Therefore,
1

(ii)

B1

M1

ln 1

ln 1

0,

ln 2

Question1
Notethatthestatementofthequestionrequirestheuseofaparticularmethodinbothparts.
(i)
B1
M1
A1
B1

Correctdifferentiationoftheexpression.
Considerationofthesignofthederivativeforpositivevaluesof .
Deductionthatthederivativeispositiveforallpositivevaluesof .
Clearexplanationthat
ln 1
ispositiveforallpositive .
Notethatanswerisgiveninthequestion.

B1

Useof

andsummation.

M1 Manipulationoflogarithmicexpressiontoformdifference.
M1 Attempttosimplifythesum(somepairscancelledoutwithinsum).
A1 Clearexplanationofresult.
Notethatanswerisgiveninthequestion.

(ii)
B1 Correctdifferentiationoftheexpression.
M1 Considerationofthesignofthederivativefor0
1.
A1 Deductionthatthederivativeisnegativeforthisrangeofvalues.
B1 Deductionthat
ln 1
isnegativeforthisrangeofvalues.
B1 Useof
andsummation.
M1 Expressionwithinlogarithmasasinglefractionandnumeratorsimplified.
M1 Logarithmsplittochangeatleastoneproducttoasumoflogarithmsoronequotientasa
differenceoflogarithms.
A1 Completesplitoflogarithmtorequiredform.
M1 Useofdifferencestosimplifysum.
A1 ln 2correct.
B1 Correctlydealingwithlimitas .
Notethatanswerswhichuseastheupperlimitonthesumfromthebeginningmust
haveclearjustificationofthelimit.Thosebeginningwith astheupperlimitmusthave
ln
ln
1 correctinsimplifiedsum.
A1 Inclusionof
1tothesumtoreachthefinalanswer.
Notethatanswerisgiveninthequestion.

Question2

sin
sin

B1
M1A1

sin
sin 3

3
3

M1
2 sin cos

(or

M1M1

A1

B1

3
3

4 sin

cos 2

4 sin

2 1

Therefore
3
1
1
3

3
3
So trisectstheangle

sin

,so

B1

sin 3

sin
sin cos 2
cos sin 2

sin
cos
sin
1 2 sin
sin
3 4 sin

B1B1

2 sin

M1M1
A1

B1
M1A1

M1M1
A1

Question2
B1
M1
A1
B1
M1
M1
M1
A1

B1
B1
B1
M1
M1
A1

B1
M1
A1
M1

(maybeimpliedbylaterworking).
Expressionfor
Applicationofthesinerule.
Correctstatement.
Doesnotneedtobestatedaslongasimpliedinworking.
Useofsin
formula.
Useofdoubleangleformulaforsin.
Useofdoubleangleformulaforcos.
Simplificationofexpression.
Notethatanswerisgiveninthequestion.

Identificationofthisrelationshipbetweendistances.(just
Correctexpressionsubstitutedforthelengthof .
Correctexpressionsubstitutedforthelengthof .
Useofdoubleangleformulaforcos.
Simplificationofexpressionobtained.
Correctexpressionindependentof .

Identificationofarightangledtriangletocalculatesin
.
Deductionthatoneofthelengthsinsineofthisangleisequalto
Valueoftheangle(Degreesorradiansarebothacceptable).
Obtaining
2

M1 Useof

A1 Expressiontoshowthat

andconclusionstated.

issufficient)

Question3

isthenumberoftrianglesthatcanbemadeusingarodoflength8and
twoother,shorterrods.
Ifthemiddlelengthrodhaslength7thentheotherrodcanbe1,2,3,4,5or6.
Ifthemiddlelengthrodhaslength6thentheotherrodcanbe2,3,4or5.
Ifthemiddlelengthrodhaslength5thentheotherrodcanbe3or4.
2 4 6.

Assumethatthelongestofthethreerodshaslength7:
Ifthemiddlelengthrodhaslength6thentheotherrodcanbe1,2,3,4or5.
Ifthemiddlelengthrodhaslength5thentheotherrodcanbe2,3or4.
Ifthemiddlelengthrodhaslength4thentheotherrodmustbe3.
Therefore
1 3 5.
1 2 3 4 5 6.

2 4 2
1
1 2 3 2
1

3.(Thepossibilitiesare 1, 2, 3 , 1, 3, 4 and 2, 3, 4 .)
Substituting

2intotheequationgives

Thereforetheformulaiscorrectinthecase
Assumethattheformulaiscorrectinthecase

2 2

1 4

3.

2.

3
1 12
4
statementoftheformulawhere
Therefore,byinduction,

2
1 4

Therefore
1 4

5 .(Or

M1
4

1 ,whichisa

1.
1 4

M1
A1

M1
M1

M1
A1
A1

B1
B1

B1

M1

1 4

M1

B1

M1

1
1

M1

A1

1 .

M1A1

1 .
1 4

1 )

A1

Question3
M1
M1
M1
A1
M1
M1
M1
A1
A1
B1
B1
B1
B1
M1
M1
M1
A1
M1
A1
A1

.
Appreciationofthemeaningof
Identifythenumberofpossibilitiesforthelengthofthethirdrodinonecase.
Identifythesetofpossiblecasesandfindnumbersofpossibilitiesforeach.
Clearexplanationoftheresult.
Notethatanswerisgiveninthequestion.
Anattempttoworkout
.
Correctcalculationforanyonedefinedcase.
Identificationofacompletesetofcases.
Correctvaluefor
.
Correctdeductionofexpressionfor
.
Correctexpression.Nojustificationisneededforthismark.
Correctexpression.Nojustificationisneededforthismark.
Correctjustificationthat
3.Requiressightofpossibilitiesorotherjustification.
Evidenceofcheckingabasecase.(Acceptconfirmationthat
1gives
0here.)
Applicationofthepreviouslydeducedresult.
Substitutionofformulafor
andtheformulaforthesum.
Takingcommonfactortogiveasingleproduct.
Rearrangementtoshowthatitisastatementoftherequiredformulawhen
1
andconclusionstated.
Useofresultfromstartofquestion.
Correctsummationof2 4 2
1 .
Correctformulareached(anyequivalentexpressionisacceptable).

Question4
(i)

B1
B1

(ii)

,sostationarypointswhen

1.
B1
B1
M1
A1
A1

B1
B1
B1
B1

(iii)
B1
B1
B1
B1

B1
B1
B1
B1
B1


Question4
Penaliseadditionalsectionstographs(verticaltranslationsby
providingthatthecorrectsectionispresentinlaterparts.

)onlyonthefirstoccasion

B1
B1

Correctshape.

B1
B1
M1
A1
A1

B1
B1
B1

Rotationalsymmetryaboutthepoint 0,0 .
Correctshape.
Differentiationtofindstationarypoints.
.( coordinates)
Correctstationarypoints 1,
Correct coordinates.

Rotationalsymmetryaboutthepoint 0, .
Correctshape.
Stationarypointshavesame coordinatesaspreviousgraph.(Followthroughincorrect
stationarypointsinpreviousgraphifconsistenthere).
Correctcoordinatesforstationarypoints 1,
arctan

Correctasymptotes
1.
axisasanasymptote.
Middlesectioncorrectshape.
Outsidesectionscorrectshape.

Sectionfor 1
1correctshape.
1
.

B1

B1
B1
B1
B1

B1
B1
B1
B1
B1

Asymptotes

and

shown.

Sectionfor
Sectionfor
0, .

1correctwithasymptote
1correctwithasymptote

2 .
0orarotationof

1sectionabout

Question5
(i)

tan

tan arctan

arctan

M1

tan

tan

,whichsimplifiesto

tan

M1

M1A1

Hence,byinductiontan

Supposethatitisnottruethat

arctan

Clearly,

A1

B1

.
arctan

forallvaluesof .
arctan

Thenthereisasmallestpositivevalue, suchthat

Since

arctan

andtan

However,

Therefore

arctan
arctan

M1M1
A1

tan 2

M1A1

So,

Whichsimplifiesto2 tan
tan
2 tan
2
Since mustbeacute,tan

B1

arctan

Therefore
lim

tan
1 4
1
0
cannotequal 2 .

arctan 1

M1A1
A1
B1

A1

,sothisisnotpossible.

.
arctan

,but

(ii)

B1

Assumethattan

1.

,sotheformulaiscorrectfor

M1A1

Question5
Confirmationthattheformulaiscorrectfor
1.
Expressionof
intermsof .
Useoftan
formula.
Simplificationoffraction.
Expressionof
toshowthatitmatchesresult.
Conclusionstated.

Confirmationfor
1.
Observationthat
0
Evidenceofunderstandingthatsuccessivevaluesof withthesamevalueoftan must
differby .
A1 Evidenceofunderstandingthat
cannotbesufficientlylargefor tobeofthe
formarctan if
is.
A1 Clearjustification.

M1 Identificationoftherelevantsidesofthetriangle(diagramissufficient).
A1 Correctexpressionfortan 2 .
B1 Useofdoubleangleformula.
M1 Rearrangementtoremovefractions.
A1 Correctquadraticreached.
A1 Quadraticfactorised.
B1 Irrelevantcaseeliminated(mustbejustified).
M1 Sumexpressedaslimitof
A1 Correctvaluejustified.
Notethatanswerisgiveninthequestion.

B1
M1
M1
M1
A1
A1

B1
M1
M1

Question6
(i)

sec

cos cos
2

sec

cos

cos

sin

Therefore,sec

(ii)

sin

sin

M1
M1A1

tan

M1A1

1
sin

B1
sin
sin

Therefore,

sin
sin

sin
sin

sin

B1

sin

sin

So,2

M1

A1

,andapplyingtheresultfrompart(i):
tan

Consider

tan

So,

Therefore,

sin

sin
sin
2

Andso,

Therefore,

2.

.Makingthesubstitution

B1

sin

M1A1

sin

sec

sec

tan
tan

sec

B1
M1
A1
B1

B1

1
sin

2 tan

tan

sin

2 sin cos

(iii)

Limits:

B1

sin

Therefore,

sin sin

cos

B1

sec

cos

B1

Question6
B1
B1

Expressionofsec intermsofanyothertrigonometricfunctions.
Correctuseofaformulasuchasthatforcos
toobtainexpressionwith
trigonometricfunctionsof .
M1 Expandingthesquaredbrackets.
M1 Useofsin 2 sin cos andsin
cos
1
A1

M1
A1
B1

B1
M1
A1
B1
B1

M1
A1
B1
M1
A1
B1
B1

Fullyjustifiedanswer.
Notethatanswerisgiveninthequestion.

.
Anymultipleoftan
Correctanswer
Dealswithchangeoflimitscorrectly.AND
Correctlydealswithchangetointegralwithrespectto .
Notethatboththesestepsneedtobeseenthecorrectresultreachedwithoutevidence
ofthesestepsshouldnotscorethismark.
Useofsin
sin (maybejustseenwithinworking)
Groupingsimilarintegrals.
Fullyjustifiedanswer.
Notethatanswerisgiveninthequestion.
Evaluationoftheintegralfrom(i)withtheappropriatelimits.
Useofresultfrom(ii)toevaluaterequiredintegral.

Attempttomakethesubstitution.
Substitutionallcompletedcorrectly.
Rearrangetogivesomethingthatcanrepresenttherequiredintegralononeside.
1 tan withinintegral.
Useofsec
Correctevaluationofthisintegral.
Correctuseofresultfrompart(i).
Correctapplicationofresultdeducedearliertoreachfinalanswer.

Question7
(i)

(ii)

Mostlikelyexamples:

and

If
thentherecannotbetwopointsonthecirclethatareadistanceof2
apartandanytwodiametricallyoppositepointson mustbeadistanceof2
apart.
If
thenthecirclemustbethesameas ,sothereisnotexactly2points
ofintersection.

Thedistancesofthecentreof fromthecentresof and are

and
.
Ifthe coordinateofthecentreof is ,thenthe coordinateisgivenby
and

Therefore,
4

andso

Therefore,the coordinateofthecentreof satisfies

16
16
16
16
16
16

M1A1
B1
B1B1
M1

B1

So,

Therefore,
8
16
4
4
4
4

16

B1
0

8
4
2
4

B1

So2

B1

M1A1

and

M1M1
A1

M1M1

M1
M1

A1

Question7
M1
M1
A1
B1
B1

M1
A1
B1
B1
B1
M1
M1
A1

B1

B1

M1
M1
M1
M1
A1

.
Calculationthatthedistancebetweenthecentresofthecirclesmustbe
Anexamplewhichshowsthatitispossibleforatleastonevalueof .
Exampleshowingthatitispossibleforall
.
Statementthatthetwointersectionpointsmustbeadistance2 apart.
Explanationthatinthecase
itwouldhavetobethesamecircle.

Thelinejoiningthecentreof (or )andtheradiitoapointofintersectionformaright


angledtriangleineachcase.(onecase)
Useofthistofindthedistancebetweencentresofcircles.
Applyingthesametotheothercircle.
Expressionrelatingthecoordinatesandradiiobtainedfromconsidering .
Expressionrelatingthecoordinatesandradiiobtainedfromconsidering .
Eliminationof fromtheequations.
Eitherexpansionofsquaredtermsorrearrangementtoapplydifferenceoftwosquares.
Expressionfor reached.
Notethatanswerisgiveninthequestion.

Substitutiontofindexpressionfor coordinate.
Notethatanyexpressionfor intermsof , , and issufficient,butitmustbe
.
expressedas
,not
Observationthat mustbepositive.
Alternativemarkschemeforthismayberequiredoncesomesolutionsseen.
ontheleft.
Attempttorearrangetheinequalitytoget16
Reachapointsymmetricin and .
Reachacombinationofsquaredterms.
Applydifferenceoftwosquarestosimplify.
Reachtherequiredinequality.
Notethatanswerisgiveninthequestion.

Question8
(i)

Let bethevectorfromthecentreof to .
Usingsimilartriangles,thevectorfromthecentreof to is

Therefore

fromthecentreof tothecentreof

So

Thepositionvectorof is

(ii)

M1A1

,sincethesearebothexpressionsforthevector
.

A1

Thepositionvectorsof and willbe

M1A1

and

B1

Therefore,

Similarly,

Sincetheyaremultiplesofeachotherthepoints , and mustlieonthe


samestraightline.

lieshalfwaybetween and if

Therefore

So,
Whichsimplifiesto

M1A1
M1A1

(iii)

M1

M1A1
M1A1
B1

B1
M1

M1A1

Question8
M1
A1
M1
A1
M1
A1

B1
M1
A1
M1
A1
M1
A1
M1
A1
B1

B1
M1
M1
A1

Identificationofsimilartriangleswithinthediagram.
Relationshipbetweenthetwovectorsto .
Equatingtwoexpressionsforthevectorbetweenthecentresofthecircles.
Correctsimplifiedexpression.
Calculationofvectorfromcentreofonecircleto .
Correctpositionvectorfor .
Notethatanswerisgiveninthequestion.

Identifyingthecorrectvectorsforthefocioftheotherpairsofcircles.
Expressionforvectorbetweenanytwoofthefoci.
Termsgroupedbyvector.
Simplificationofgroupedterms.
Extractionofcommonfactor.
Expressionforavectorbetweenadifferentpairoffoci.
Awardmarksassameschemeforpreviousexample,butawardallfourmarksforthe
correctanswerwrittendownasitcanbeobtainedbyrotating1,2and3intheprevious
answer.
Statementthattheylieonastraightline.

Statementthatthetwovectorsmustbeequal.
Reductiontostatementinvolvingonly terms.
Attempttosimplifyexpressionobtained(ifnecessary).
Anysimplifiedform.

Question9
(i)

Takingmomentsabout :
3
sin 30

5
sin 30

5
cos 30 sin
cos sin 30
3
cos 30 sin
cos sin 30
1
1
3
3
sin
cos
3
sin
cos
5
2
2
2
2
Therefore
cos
43 sin
Either
Usesin
cos
1andjustifychoiceofpositivesquareroot.
Or
Drawrightangledtrianglesuchthattan
andcalculatethelengthofthe

M1A1
M1A1
B1
M1
A1
A1

M1

hypotenuse.

(ii)

sin

A1

Let betheverticaldistanceof below .


Let betheverticaldistanceof below .

sin

M1M1
A1

sin

M1A1

If isthecentreofmassofthetriangle:

M1A1

Conservationofenergy:
4
8 . 2 forcompleterevolutions.

M1A1

Therefore

A1

Question9
M1
A1
M1
A1
B1
M1
A1
A1
M1
A1

M1
M1
A1
M1
A1
M1
A1
M1
A1
A1

Attempttofindthemomentof about .
Correctexpressionformoment(sin 30
maybereplacedbycos 60
).
Attempttofindthemomentof about .
Correctexpressionformoment(sin 30
maybereplacedbycos 60
).
Correctstatementthatthesemustbeequal.
Useofsin
orcos
formulae.
Correctvaluesusedforsin 30andcos 30.
Correctlysimplified.
Useofacorrectmethodtofindthevalueofsin .
Fullyjustifiedsolution.Ifusingrightangledtrianglemethodthenchoiceofpositiverootnot
needed,ifchoiceofpositiverootnotgivenwhenapplyingsin
cos
1method
thenM1A0shouldbeawarded.
Notethatanswerisgiveninthequestion.

Attempttofind .
Correctlydealwithsineorcosineterm.
Correctvalue.
Attempttofind .
Correctvalue.
Combinetwovaluestoobtaindistanceofcentreofmassfrom .
Correctvalue
Applyconservationofenergy.
Correctinequality.
Correctminimumvalue.

Question9Alternativepart(i)
(i)

Let bethecentreofmassofthetriangleandletthedistance
Takingmomentsabout :
5
cos
3
cos
Therefore5
3
,so
.
mustlieon and
30
.
cos

sin 30
sin 30 cos

cos 30 sin

43 sin andsocos

,andso(since isacute)sin

48 sin

Takingmoments.
Correctequation.
Correctrelationshipbetween and .
Identificationthat lieson andcalculationof
.
Useofsineofidentifiedangle.
Useofsin
formula.
Directrelationshipbetweensin andcos .
Rearrangementandsquaringbothsides.
cos
1.
Applyingsin
Finalanswer(choiceofpositiverootmustbeexplained).
Notethatanswerisgiveninthequestion.

M1A1
A1
B1
M1

A1

M1
A1
A1
B1
M1
M1
A1
M1
M1
A1

M1

Thereforecos
sin

cos

be .

M1
M1A1

Question10

Ifthelengthofstringfromtheholeatanymomentis ,then

Thedistance, ,fromthepointbeneaththeholesatisfies,
2

Therefore

,and

Therefore,thespeedoftheparticleis cosec .

sin

Therefore

Theaccelerationis

Since

Horizontally:

A1

cosec cot

M1A1

A1
M1

sec ,theaccelerationcanbewrittenas

cot

M1A1

cot

,so

cot

andsotan

M1M1
A1

cosec

Theparticlewillleavethefloorwhen cos
cot

M1

cot

M1

cot

B1
M1A1

,socos

sin

cosec

Acceleration:

cosec

B1

M1A1
M1A1

Question10
B1
B1
M1
A1
M1
A1

Aninterpretationof intermsofothervariables(includinganynewlydefinedones).
Anyvalidrelationshipbetweenthevariables.
Differentiationtofindhorizontalvelocity.
Correctdifferentiation.
Attempttoeliminateanyintroducedvariables.
Correctresult.
Answerswhichmakeclearreferencetothespeedoftheparticleinthedirectionofthe
stringbeingV.

M1 Differentiationofspeedfoundinfirstpart.
A1 Correctanswer.
M1 Attempttodifferentiatetofindanexpressionfor .
A1
M1
M1
A1

M1
M1
A1

M1
A1
M1
A1

Correctanswer.
Substitutiontofindexpressionforacceleration.
Relationshipbetweenrequiredvariablesandanyextravariablesidentified.
Substitutiontogiveanswerintermsofcorrectvariables.

Horizontalcomponentoftension.
ApplicationofNewtonssecondlaw.
Correctanswer.

Verticalcomponentoftensionfound.
Identificationthatparticleleavesgroundwhentensionisequaltothemass.
Substitutionoftheirvaluefor .
Rearrangementtogiverequiredresult.
Notethatanswerisgiveninthequestion.

Question11
(i)

(ii)

cos , sin
Differentiating:
sin
, cos

Since ismovingwithvelocity andisatthepoint , 0 attime ,


:
sin ,
cos .
Velocityof is

Initialmomentumwas (horizontally).
Horizontalvelocityof willbethesameasthatof ,sohorizontallythetotal
momentumisgivenby
2
sin
Therefore3
2 sin
.

Initialenergywas

Totalenergyis

Therefore
3
So

(iii)

(iv)

A1

2
2

sin

cos

M1A1

sin

cos

M1

gives
2

3
4

sin
sin

4 sin
4
sin
4
2

2 sin
sin
4

M1

sin

sin

A1

So,

0,sotherecanonlybeaninstantaneouschangeofdirectioninwhich
variesatacollision.Sincethefirstcollisionwillbewhen
0,thesecond
collisionmustbewhen
.

Sincehorizontalmomentummustbe ,
0 2 sin
.
TheKEof mustbe

,so

sin

isonly0when takesthesevaluesand ispositiveas wouldneedanon


zerovaluetosatisfy3
2 sin
if isnegative.(Therelationshipisstill
truesincecollisionsareelastic).

sin
,so

or

sin

Substituting
3
6
6

M1

M1

2
sin

B1B1
M1

A1

M1

B1
B1

B1
B1

M1A1
B1

Question11
B1
B1
M1
A1

M1
M1
A1
M1
M1
A1
M1
M1
A1

B1
B1

B1
B1
M1
A1
B1

Horizontalcomponent.
Verticalcomponent.
Differentiation.
Completejustification,includingclearexplanationthat
.
Notethatanswerisgiveninthequestion.

Statementthatmomentumwillbeconserved.
Identificationthathorizontalmomentumof and willbeequal.
Correctequationreached.
Notethatanswerisgiveninthequestion.
Statementthatenergywillbeconserved.
Useofsymmetrytoobtainenergyof (acceptanswerswhichsimplydoubletheenergyof
ratherthanstatingtheverticalvelocityinoppositedirection).
Correctrelationship.
cos
1.
Useofsin
Substitutingtheotherrelationshiptoeliminate .
Correctequationreached.
Notethatanswerisgiveninthequestion.

Correctvalueof .
Answerjustified.

Firstequationidentified.
Secondequationidentified.
Solvingsimultaneouslytofind .
Correctvaluesfor .
Justifiedanswerthat isnotalways0when takesthesevalues.

Question12
(i)

Ifatailoccursthenplayer mustalwayswinbefore canachievethe


sequencerequired.Thereforetheonlywayfor towinisifbothofthefirst
twotossesareheads.
Afterthefirsttwotossesareheadsitdoesnotmatterifmoretossesresultin
headsasthefirsttimetailsoccurs willwin.
Theprobabilitythat winsistherefore

B1
B1

(ii)

Asbefore,after ,only canwin.


Similarly,after ,only canwin.
Inallothercasesforthefirsttwotossesonly and willbeabletowin.
Theprobabilitiesfor and towinmustbeequal.

B1
B1
M1
M1

Theprobabilityofwinningis foralloftheplayers.

A1

(iii)

Ifthefirsttwotossesare then mustwin(assoonasa occurs),sothe


probabilityis1.
After :
mustwinifthenexttossisa as needstwo stowin,but willwinthe
nexttimean occurs.
Ifthenexttossis ,thenthepositionisasifthefirsttwotosseshadbeen ,
andsotheprobabilitythat winsfromthispointis .

B1
M1
M1

Therefore,

After :
Ifthenexttossis then willwinwithprobability .
Ifthenexttossis then willwinwithprobability .

M1

,andso

A1

Therefore

A1

After :
Ifthenexttossis thenplayer winsimmediately.
Ifthenexttossis then willwinwithprobability .
Therefore

M1

A1

Fromthethirdequation

Solvingthetwoequationsin and ,gives

M1A1

Theprobabilitythat winsis

B1

M1A1

Question12
B1
B1
B1

B1
B1
M1
M1
A1

B1
M1
M1
A1

M1
A1

M1
A1

M1
A1
M1
A1

Identifyingthat cannotwinonceatailhasbeentossed.
Identifyingthat mustwinoncethefirsttwotosseshavebeenheads.
Showingthecalculationtoreachtheanswer.
Notethatanswerisgiveninthequestion.

Recognisingthatthesituationisunchangedforplayer .
Recognisingthatthesamelogicappliestoplayer .
Allothercasesleadtowinsforoneoftheremainingplayers.
Recognisingthattheprobabilitiesmustbeequal.
Correctstatementoftheprobabilities.
Ifnomarkspossiblebythisschemeawardonemarkforeachprobabilitycorrectly
calculatedwithsupportingworking.Allfourcalculatedscores5marks.

Explanationthatprobabilitymustbe1.
Explanationofthecasethatthenexttossis .
Thismarkandthenextcouldbeawardedforanappropriatetreediagram.
Explanationofthecasethatthenexttossis .
Justificationoftherelationshipbetween and .
Notethatanswerisgiveninthequestion.

Considerationofonecasefollowing .
Thismarkcouldbeawardedforanappropriatetreediagram.
Establishmentoftherelationship.

Considerationofonecasefollowing .
Thismarkcouldbeawardedforanappropriatetreediagram.
Establishmentoftherelationship.

Attempttosolvethesimultaneousequations.
Correctvaluesfor , and .
Attempttocombineprobabilitiestoobtainoverallprobabilityofwin.
Correctprobability.

Question13

(i)

for
for

B1

M1M1
A1

Usethesubstitution

intheintegral:

B1

Therefore

If

1thenchoose

If

1thenchoose

(ii)

.
ln .

ln asitispositive.

Applyingintegrationdonebefore:
2
2

intheintegral:
2

M1A1

and,applyingtheintegrationalreadycompleted,

.
Therefore

Var

Var

.
2

Var
For

A1

0,

1
0,sothevariancedecreasesas increases.

Var

M1

Var

A1

M1A1

B1

Usingintegrationbyparts:

B1

M1A1

0astheminimumoccursatanegativevalueof .

Usethesubstitution

A1

,sothestationarypointoccurswhen

M1

M1
A1

Question13
B1 Statementofrandomvariable.
M1 Anycorrectterminexpectation(allow multipliedbyanattemptattheprobabilityfor
notneedinganyextracosts).
M1 Correctintegralstated(allow missing).
A1 Fullycorrectstatement.

Maybealteredtoaccommodateothermethodsoncesolutionsseen.
B1 Substitutionperformedcorrectly.
M1 Integrationbypartsusedtocalculateintegral.
A1 Correctlyjustifiedsolution.
Notethatanswerisgiveninthequestion.
M1 Differentiationtofindminimumpoint.
A1 Correctidentificationofpoint.
B1 Bothcasesidentifiedwiththesolutionsstated.

M1 Attemptat
(atleasttwotermscorrect).
A1 Correctstatementof
.
B1 Substitutionperformedcorrectly.
M1 Applyingintegrationbyparts.
A1 Correctintegration.
A1 Correctexpressionfor
.
M1 UseofVar

A1 CorrectsimplifiedformforVar

M1 DifferentiationofVar
.
A1 Correctinterpretation.
Notethatanswerisgiveninthequestion.

1.(i)
1

1
1

B1
1
1

1
2

integratingbyparts

M1A1

A1*

(4)

(5)

1
2

1
1

M1

M1

B1

tan

Thus

M1
A1*

(ii)
.

usingthesubstitution

andthenthesubstitution

So
Usingthesubstitution

1 M1A1*

M1A1*

(iii)

M1A1

M1

M1

A1(5)

So

!
!

M1A1*(6)

2.(i)True.

B1

B1

1000

If

1000,then1000

(ii)False.

1and

E.G.Let

2for odd,and

Then forwhichfor

Soitisnotthecasethat
(iii)True.

,i.e. 1000

,so1000

B1
2and

,nor

1for even.

B1

M1

,butnorisitthecasethat

M1A1(4)

A1(4)

B1

meansthatthereexistsapositiveinteger,say

,forwhichfor

meansthatthereexistsapositiveinteger,say

,forwhichfor

E1

E1
max

Thenif
for

B1

A1(5)

B1

B1

4.

B1

M1A1

B1

sobytheprincipleofmathematicalinduction,

2 forsomevalue

Assume
Then

,andso

(iv)True.

2 for

4,andthus

2 A1(7)

3.(i)

If|
So

Symmetryaboutinitialline

Twobranches

Shapeandlabelling

sec |
,then
sec
sec

sec

or

G1
G1
G1(3)
or
sec

M1A1

Ifsec
0, sec
0as
and sec
arebothpositive,andthusinbothcases,
0whichisnotpermitted.
Ifsec

0, sec

sosec

0asrequired.

So
sec
thepole(origin),
dbeingb,

0and sec

B1

0giving

0as and
B1
0

(4)

,thuspointssatisfying(*)lieonacertainconchoidofNicomedeswithAbeing

B1

sec .

andLbeingtheline

B1

B1(3)

(ii)

Symmetryaboutinitialline
Twobranches

Loop,shapeandlabelling

G1
G1
G1


If

sec

,thenthecurvehastwobranches,

0,theendpointsoftheloopcorrespondingtosec

withsec

0and

withsec
.

B1(4)

1and

Inthecase

2so

2,sec

Areaofloop
2

sec
sec

4 sec
3

4 ln

4
2

tan

M1A1
4 ln|sec
3

tan |
4 ln 2

4
3

M1A1
M1A1(6)

sec

4.

(i)

For

andfor ,

iscontinuous.
.

B1

Sothesketchofthisgraphmustbeoneofthefollowing:

B1
Hence,itmustintersectthe axisatleastonce,andsothereisatleastonerealrootof
0

B1(3)
M1

(ii)
Thus

A1

0,

and,as

A1

0,

addingthesethreeequationswehave,
3

0 M1

(Alternatively,

So
Thus6

M1

A1*(6)

(iii)

cos

Let

cos 2

Then
As

sin

for

sin 2

and

1, 2, 3

cos 3

sin

M1
sin 3

bydeMoivre

M1

sin 2

sin 3

0
andso , ,and arereal,

M1

andthereforesoare , ,and

A1
0with , ,and real,bypart

Hence,as , ,and aretherootsof


(i),atleastoneof , ,and isreal.
cos

Soforatleastonevalueof ,
andas
If

M1
sin

0asrequired.

A1(6)

0(say
and

0)
andsothequadraticofwhich and arethe

rootshasrealcoefficients.Thus ,
If

Thuscos
Butsin
If

0then isreal. and aretherootsof

whichis

0,

cos

,andso

sin

andso

0,

isrealandthus,sin

.(

,as
.

0because

0)

M1

M1A1

0,andthus
B1(5)

B1

0,then and arerealroots,sosin


.

sin

0,so

,where ,

5.(i)Havingassumedthat2isrational(step1),2

0B1

,so provingstep3.

Thusfromthedefinitionof (step2),as and2


B1(2)
If ,then isanintegerand 2isaninteger.
1

So 2

isaninteger,

and 2

2andsoM1

B1
B1

2whichisanintegerandso 2

1,andthus0

provingstep5.B1(3)

A1

andthusthiscontradictsstep4that isthesmallestpositiveintegerin as 2
showntobeasmallerpositiveintegerandisin .

A1(3)

(ii)

If2

So 2

,thatis2

andhence2

andso2

,whichcanbewritten2

,where ,

0
2

M1

0M1

isrationalandthat2

provingthat2

hasbeen

isrational. A1

provingthat2

isrational,then2

If2

,where ,

isrational,then2

isrationalonlyif2

isrational.

A1(4)
Assumethat2

isrational.

Definetheset tobethesetofpositiveintegerswiththefollowingproperty: isin ifandonlyif


2

and 2

areintegers.

B1
,where ,

Theset containsatleastonepositiveintegerasif2
2

and

,so

Define tobethesmallestpositiveintegerin .Then 2


Consider

1 .

itselfaninteger.
and

1
2

0,then

M1A1

areintegers. B1

whichisthedifferenceoftwointegersandsois

2
2

and 2

2
2

whichisaninteger,
2

whichisaninteger.

Thus

1 isin .
2andso0

M1A1
1

1,andthus0

that isthesmallestpositiveintegerin as
integerandisin .

M1A1(8)

,andthusthiscontradicts

1 hasbeenshowntobeasmallerpositive

6.(i)

For ,

B1

,werequiretosolve

M1

2
2

0
8

2
2

4
2
2

Sofor,

,as

i.e.

so3

soif

,then 2

,
1

1or

Thus
Soas

0,

M1

M1A1

For ,

,from(i)werequire
.

inotherwords

1inwhichcase 1

whichitis,

1,

thediscriminantis

,so
0so

3,i.e.

M1A1(12)
whichitis,and

M1

So and neednotbereal.Acounterexamplewouldbe
forthen

M1A1

0,thevaluesof arereal.B1

and

Therearethreedistinctvaluesof unless

M1A1

Thusif
Thus

M1A1
3

mustbereal,and2

B1*(5)
2

M1A1

mustbereal,

(ii)

,i.e.2
B1(3)

B1
2

0inwhichcase

7.

(i)
Suppose
forsomeinteger .

M1

M1A1(3)

isapolynomialofdegree i.e.

B1

Then
1

whichisapolynomialofdegree .

Suppose

1,

istruefor

whichisdivisibleby 1

Wehaveshownthatifitistruefor
foranypositiveinteger
.B1(6)

for

sotheresult

1.Hencebyinduction,itis

i.e.
B1

,itistruefor

B1(6)

M1A1

isdivisibleby 1
1.

Then

,then

andwehaveshownthatifitistruefor
trueforanypositiveinteger.

1
(ii)
Suppose
someinteger ,with

M1A1

.M1A1
1.M1A1

soresultistruefor

1.Hencebyinduction,itistrue

,itistruefor

(iii)

M1

So

1
Butby(ii),
thusif
1,

isdivisibleby 1
1
0,andhence

1
andso

M1A1*(5)

M1A1
1

,and

cos

8.(i)

sin

sin

and

cos

Thus

sin

0,

cos

cos

sin

M1A1

sin

becomes

Thatis sin
as

cos

M1A1
cos

sin

sin
cos

sin

cos

cos

M1

Multiplyingoutandcollectingliketermsgives
cos
whichis

0.

So

andthus

sin

sin

M1

A1

A1

cos

M1A1*(7)

G1(4)
M1soln| |

(oralternatively

A1andhence

A1)

(ii)

becomes

sin

cos

cos

sin

cos

sin

cos

sin

sin

thatis
sin

cos

cos

cos

sin

sin

cos

Multiplyingoutandcollectingliketermsgives
cos

cos

sin

0.

whichis

sin

sin

cos

0 M1

A1

So
ln

A1

A1

M1

So
1

with

0
1
1

thatis

A1*

G1G1G1(9)

9.

Iftheinitialpositionof is,thenattime ,
1
2

1
2

,soconservingenergy,

M1A1A1

Thus,

M1

i.e.

Thegreatestvalue, ,attainedby ,occurswhen

A1*(5)

0.

M1

Thus
So

(negativerootdiscountedasallquantitiesarepositive)

Thus
2

and
2

M1A1(3)

As

differentiatingwithrespectto
2

1
2

2
M1A1

Thus

A1

Sowhen

,theaccelerationof is
2

M1A1(5)

Thatis

andthus
1

where istheperiod.

M1A1

So
1

1
Let

B1

then

andso

2
as

Thus
1

M1A1

andso
4

1
asrequired.

32

M1A1*(7)

10.

Thepositionvectoroftheupperparticleis
sin
cos

B1B1

sodifferentiatingwithrespecttotime,itsvelocityis
cos
sin

E1*(3)

Itsacceleration,bydifferentiatingwithrespecttotime,isthus
sin
cos

cos
sin

M1A1A1

sobyNewtonssecondlawresolvinghorizontallyandvertically
sin
cos

sin
cos

cos
sin

M1A1

Thatis
cos
sin

sin
cos

sin
cos

Theotherparticlesequationis
sin
cos

cos
sin

sin
cos

B1(6)

Addingthesetwoequationswefind
2
i.e.

0and

M1A1*

Thus
cos
sin
i.e.

cos

sin

sin and

sin
cos
sin

sin
cos
cos

cos

Multiplyingthesecondofthesebysin andthefirstbycos andsubtracting,

0andso

Thus

0.

andasinitially2

Thereforethetimetorotateby
As

andinitially

M1A1*(4)
,

isgivenby

,attime,

therodisinitially abovethetable.

M1A1
,so

,andso

A1
asthecentreof

M1A1

Hence,giventheconditionthattheparticleshitthetablesimultaneously,
0
/
1/2
/

Hence0

,or2

asrequired.M1A1*(7)

11.(i)Supposethattheforceexertedby ontherodhascomponents perpendiculartotherod


and paralleltotherod.Thentakingmomentsfortherodaboutthehinge,
0,M1
0yields

whichas
A1*(2)

0andhencetheforceexertedontherodby isparalleltotherod.

sin

Resolvingperpendiculartotherodfor ,
sin ,

Dividingby
cot

Thatis

sin

sin

cos

M1A1

cot

cos orinotherwords cot

cos asrequired.

M1A1*(4)

Theforceexertedbythehingeontherodisalongtherodtowards , B1
andifthatforceis ,thenresolvingverticallyfor , cos
sec .

so

M1A1

A1(4)

(ii)Supposethattheforceexertedby ontherodhascomponent perpendiculartotherod


towardstheaxis,thattheforceexertedby ontherodhascomponent perpendiculartothe
rodtowardstheaxis,

B1
thenresolvingperpendiculartotherodfor
M1A1
andsimilarlyfor

sin

sin

sin

sin

cos

cos

M1A1
0

Takingmomentsfortherodaboutthehinge,
Somultiplyingthefirstequationby
sin

sin
sin ,

Thatis cot

cos ,where

M1A1

andaddingwehave

sin

Dividingby

,thesecondby

cos

sin

cot

cos

M1A1

A1(10)

cos

12.(i)Theprobabilitydistributionfunctionof is

1
6

1
6

1
6

1
6

1
6

1
6

sotheprobabilitydistributionfunctionof

is

1
6

1
6

1
6

1
6

1
6

1
6

andthus

B1

Theprobabilitydistributionfunctionof is

2
1

36

3
2

36

4
3

5
4

36

36

6
5

36

36

36

10
3

36

36

11
2

12
1

36

M1
sotheprobabilitydistributionfunctionof

0
6

is

1
6

36

2
6

36

3
6

36

4
6

36

36

5
6

36

A1
whichisthesameasfor

andhenceitsprobabilitygeneratingfunctionisalso

Therefore,theprobabilitygeneratingfunctionof
andthustheprobabilitythat
(ii)

isalso

isdivisibleby6is1 6.

B1

B1(6)

Theprobabilitydistributionfunctionof is

1
6

2
6

1
6

1
6

1
6

. A1*

36

andthus

wouldbe

Thus

M1A1

exceptthatthepowersmustbemultipliedcongruenttomodulus5.

wouldbe

except

and

1
6

1
6

So

M1A1

1
5

B1

A1

1
6

M1A1*(8)

Thatis
1
6

1
6

is 1

Wenoticethatthecoefficientof insidethebracketin

So

asrequired.

5 1

1 1

5 1
6

4.

Soif isnotdivisibleby5,theprobabilitythat
whichinturnisthecoefficientof ,namely
If isdivisibleby5,theprobabilitythat

6
6

M1
where isan

However,thiscoefficientisthesumofaGPandso
integersuchthat0

1
5 1

43

1trivially.

Thiscanbeshownsimplybyinduction.Itistruefor
Consider

1
6

35

M1A1

isdivisibleby5willbethecoefficientof
1

isdivisibleby5willbe

asrequired.
1

as

B1*

Thatis

M1A1(6)

13.(i)

G1

if0

B1

G1
2

and
0if

So

0and

if1

1if

B1(5)
1
1

1
2
2

1

2
1

0

Soas

Thus

B1

M1A1

1
2
1
2

0
1

asrequired.

(ii)

1
2

1
2
1

M1A1*(4)

2 ln

1
2

2 ln

2 ln 2

M1A1(2)

G1
1

2
1
1
2

B1(2)

Thus

So
1 1
1
2
1 1
1

2

1
2

1
2

i.e.
1
3
2
1
2 1

1
2

1
2

1
2

Soas

1
2

1
1
2

1
2

1
2

M1A1(3)

M1A1(4)

B1

1
2

1
ln 1
2
1
2

1
1
2

1
2

asrequired.

and

1
2

because,bysymmetry,

M1A1

1 1
ln
2 2

1
ln
2
1 1
ln
2 2

1 1
ln
2 2

Vous aimerez peut-être aussi